Ángulo de lanzamiento óptimo para un proyectil lanzado desde una altura sobre el suelo [cerrado]

Si un proyectil se lanza desde una altura mayor que cero y aterriza a una altura igual a cero, ¿el ángulo de lanzamiento óptimo que da el mayor alcance horizontal sigue siendo 45 grados o no?

Sé que si el proyectil cae a una altura que no es igual a la altura de lanzamiento, la fórmula

R = v 0 2 pecado 2 θ gramo
que maximiza el rango cuando el ángulo es 45 grados no es ya aplicable. Pero, ¿es esto un argumento para decir que 45 grados no es el ángulo de lanzamiento óptimo para un objeto lanzado sobre el suelo y aterrizado en el suelo? Si 45 grados no es el ángulo óptimo de lanzamiento, en este caso, ¿cuanto mayor es el ángulo menor es el alcance horizontal del proyectil?

El ángulo de lanzamiento óptimo es tal que los vectores de velocidad inicial y de aterrizaje son ortogonales entre sí. Vea mi respuesta en la pregunta vinculada.

Respuestas (5)

Si dice que el proyectil está ubicado a una altura, entonces no puede decir que 45 grados sea el ángulo de lanzamiento óptimo (sin embargo, esto sería correcto en un plano con la elevación del proyectil h = 0 unidades). Sin embargo, si el proyectil se lanza a una altura h sobre el plano, el ángulo óptimo será igual a una función F ( v 0 , h ) .

Puede calcular fácilmente el tiempo que tarda el proyectil en viajar desde A a B . Por el tiempo tomado de B a C , piensa en la energía que posee el proyectil en los dos puntos. También recuerda que el X componente de la velocidad del proyectil no cambia a lo largo del vuelo. Calcular el rango total en el X dirección y recuerda que la mayor distancia recorrida es cuando d R d θ = 0 .

Simplifica la ecuación formada y obtendrás el ángulo óptimo como función F ( v 0 , h ) .¡Parábola no a escala!

Para el tiempo tomado de A a C,

v y = v 0 s i norte ( θ )
a = gramo
Usando ecuaciones de movimiento y considerando que la dirección hacia abajo es positiva,
h = v y t + 1 2 a t 2
Resolviendo para t da
t = v 0 pecado ( θ ) gramo ( 1 + 1 + 2 gramo h v 0 2 pecado 2 ( θ ) )

componente X de la velocidad ( v 0 porque θ ) no cambia durante el vuelo.

R = v 0 porque ( θ ) t = v 0 2 pecado 2 θ 2 gramo ( 1 + 1 + 2 gramo h v 0 2 pecado 2 θ )

Ahora d R d θ = 0 para rango máximo. La diferenciación implícita es muy útil como se describe aquí .

Si dice que el proyectil está ubicado a una altura, entonces no puede decir que 45 grados sea el ángulo de lanzamiento óptimo. Estoy bastante seguro de que está equivocado al respecto. Incluso a y = h la distancia horizontal máxima recorrida permanece para 45 . OMI.
Creo que quisiste decir: θ = 1 2 porque 1 ( gramo h v 0 2 + gramo h ) . ¿Pero realmente no muestras cómo llegaste a esta simple relación?
Para puedes 'romper' las reglas. ;-)
Pero en realidad probaré el método de RWBird.
Mastermind 817: Su expresión para t B C falta ag, la expresión debajo de la raíz cuadrada en R no es adimensional, y encontrar esa derivada debe haber sido divertido.
Gert: Tenga en cuenta que la y final es cero.
Oh, sí, olvidé agregar uno en el denominador @RWBird. Nah, Wolfram Alpha me ayudó en la derivada.
Gracias, mente maestra.

Intentaré dar una respuesta cualitativa sin necesidad de matemáticas.

Si bien el ángulo de 45° brinda la distancia máxima para la misma altura, debe ajustarse para las diferencias de altura, lo que da como resultado un ángulo óptimo más plano. ¿Por qué?

Sabemos que el proyectil sigue una parábola, lo que significa que en su trayectoria descendente pasará la altura de lanzamiento con el mismo ángulo con el que fue lanzado.

La ruta se puede dividir en dos partes, la parte superior del lanzamiento y la parte inferior del lanzamiento.

Imaginemos un ángulo ligeramente más plano, por ejemplo, 44°. La parte de lanzamiento por encima seguirá haciendo casi la misma distancia que el lanzamiento de 45° (cerca de un punto óptimo, las cosas cambian lentamente), pero la continuación (la parte de lanzamiento por debajo) hará más distancia debido al ángulo más plano.

Un ángulo de más de 45° no puede dar como resultado una distancia total mayor, ya que ambas partes hacen una distancia más corta (la parte superior del lanzamiento porque estamos lejos de su óptimo de 45° y la parte inferior del lanzamiento debido al ángulo descendente más pronunciado). ).

Lo óptimo seguramente será en algún ángulo positivo hacia arriba, ya que el lanzamiento con un componente hacia abajo seguramente será peor (reduce tanto la velocidad horizontal como el tiempo de vuelo en comparación con un lanzamiento horizontal).

Entonces queda la pregunta de dónde es óptimo entre 0° y 45°. Esto solo se puede responder exactamente usando matemáticas, en algún ángulo donde la pérdida en la distancia por encima del lanzamiento ya no se compensa con la ganancia en la distancia por debajo del lanzamiento.

El ángulo para el alcance máximo desde una altura, h, no es de 45 grados. Para encontrarlo comienza con las ecuaciones de los componentes: x = v o cos(θ) t y y = h + v o sen(θ) t – (1/2)g t 2 = 0. Resuelva la ecuación x para cos(θ) y la ecuación y para sin(θ). Entonces s i norte ( θ ) 2 + C o s ( θ ) 2 = 1. Esto conduce a una ecuación cuadrática en t 2 lo que da dos valores positivos para t. Éstos corresponden a los dos ángulos posibles para alcanzar un objetivo a una distancia conocida hacia abajo. A medida que se acerca al rango máximo, los ángulos (y los tiempos) convergen. En este punto, la raíz cuadrada en la cuadrática es igual a cero. Puede establecerlo igual a cero y resolver para X 2 . Con la raíz cuadrada en cero, la cuadrática da t 2 . Combínalos en la ecuación x para obtener cos(θ).

Si h > 0 entonces, como han señalado otras respuestas, el tiempo de vuelo t es la raíz positiva de una cuadrática, y el rango R = v X t es

R ( θ ) = v 0 2 s i norte ( 2 θ ) 2 gramo ( 1 + 1 + 4 gramo h v 0 2 porque 2 θ )

dónde θ es el ángulo entre el ángulo de lanzamiento y la vertical. Encontrar el valor de θ que maximiza R es dificil pero si gramo h v 0 2 << 1 podemos aproximar R ( θ ) por

R ( θ ) v 0 2 pecado ( 2 θ ) gramo + 2 broncearse ( θ ) h

y luego

d R d θ 2 v 0 2 porque ( 2 θ ) gramo + 2 segundo 2 ( θ ) h

Cuando 0 < θ < 45 o tenemos d R d θ > 0 y en θ = 45 o tenemos d R d θ 4 h > 0 , por lo que para maximizar R ( θ ) tenemos que hacer θ mas grande que 45 o es decir, la trayectoria óptima es menos profunda que cuando h = 0 .

¿Cómo obtuviste la primera ecuación?
Pero, ¿es correcta la ecuación para el alcance en función del ángulo de lanzamiento que comenté en la respuesta anterior?
@JordanG La primera ecuación proviene de encontrar la raíz positiva de la ecuación cuadrática satisfecha por el tiempo de vuelo t y luego usando este valor de t en R = v X t . La ecuación que diste para R solo es correcto cuando h = 0 . No es correcto cuando h > 0 .
Hay un comentario anterior de Mastermind817 para la ecuación de rango que se parece a la tuya pero en realidad no es lo mismo. He seguido sus pasos y parece que lo he conseguido. Pero no estoy seguro del tuyo, si hiciste algo más correcto al derivar esa ecuación.

Sé que si el proyectil cae a una altura que no es igual a la altura de lanzamiento, la fórmula que maximiza el alcance cuando el ángulo es de 45 grados ya no es aplicable. Pero, ¿es este un argumento para decir que 45 grados no es el ángulo de lanzamiento óptimo para un objeto lanzado sobre el suelo y aterrizado en el suelo?

Trayectoria

El ángulo que proporciona la mayor R restos 45 . Pero lanzado en ese ángulo desde un elevado ( y > 0 ) la posición de lanzamiento en realidad aumenta R algo, con respecto al lanzamiento desde y = 0 .

Entonces, si quisiera alcanzar el mismo objetivo desde una posición de lanzamiento elevada, tendría que ajustar el ángulo o la velocidad inicial.


Semimatemáticamente, podemos mostrar que la distancia horizontal recorrida se da de la siguiente manera.

El tiempo pasado en el aire para el caso. y = 0 es:

t 0 = 2 v 0 pecado α gramo

Y debido a que las velocidades vertical y horizontal son independientes entre sí (invariancia galileana), la distancia recorrida horizontalmente es:

R 0 = t 0 v 0 porque α

Pero en el caso de que y > 0 Entonces el tiempo t 1 gastado en el aire es más largo y así:

R 1 = t 1 v 0 porque α > R 0

R = b b 2 4 a C 2 a
dónde a = gramo 2 v 0 2 C o s 2 θ , b = t a norte θ , C = y 0
¿Es esa la fórmula para el rango si la altura inicial está sobre el suelo?
usé la fórmula R = v 0 X t y y = y 0 + v 0 y t 1 2 gramo t 2
Si h > 0 el ángulo de lanzamiento que maximiza R no es 45 o - ver mi respuesta a continuación.